limiet

Moderators: dirkwb, Xilvo

Forumregels
(Middelbare) school-achtige vragen naar het forum "Huiswerk en Practica" a.u.b.
Zie eerst de Huiswerkbijsluiter
Gebruikersavatar
Berichten: 3.330

Re: limiet

Ik vind wel dezelfde uitkomst maar langs:
\(\int_0^1\frac{dx}{1+x}=\ln{2}\)
.

Ik heb namelijk 1/n voor de haakjes gezet en dan de formule van PeterPan toegepast.
Volgens mijn verstand kan er niets bestaan en toch bestaat dit alles?

Gebruikersavatar
Berichten: 24.578

Re: limiet

Dat is natuurlijk hetzelfde, op een verschuiving van 1 na (n = x + 1).
"Malgré moi, l'infini me tourmente." (Alfred de Musset)

Re: limiet

Er bestaat nog een heel aardig stellinkje waarmee je deze limiet aan kunt pakken:

Als
\(b_1, b_2, \cdots \)
een stijgende onbegrensde rij is en
\(\lim_{n \rightarrow \infty} \frac{a_{n+1}-a_n}{b_{n+1}-b_n} = R\)
,

dan is
\(\lim_{n \rightarrow \infty} \frac{a_n}{b_n} = R\)
.

Als we onze limiet schrijven als
\(\lim_{n \rightarrow \infty}\frac{a_n}{b_n}\)
,

dan is
\(\lim_{n \rightarrow \infty} \frac{a_{n+1}-a_n}{b_{n+1}-b_n} = \lim_{n \rightarrow \infty}\frac{\sqrt{n+1}}{(n+1)\sqrt{n+1}-n\sqrt{n}} = \lim_{n \rightarrow \infty} \frac{1}{n+1} \frac{1}{1- (1-\frac{1}{n+1})^{\frac{3}{2}}} = \frac{2}{3}\)

Gebruikersavatar
Berichten: 24.578

Re: limiet

Die stelling ben ik nog nooit tegengekomen (denk ik), toch niet in die vorm. Aardig...
"Malgré moi, l'infini me tourmente." (Alfred de Musset)

Re: limiet

Het stellinkje staat me bij, omdat het 2 maal fraai wordt toegepast in het volgende probleem:

Stel voor n :?: [rr] geldt
\(x_1=e \mbox{ en } x_{n+1} = x_{n} + e^{-x_n}\)
Bereken
\(\lim_{\nrightarrow \infty}\frac{x_{n}}{\ln(n)}\)
Het is duidelijk dat \((x_n)\) een stijgend rijtje is.

Als \(\lim_{\nrightarrow \infty}x_n = a\), dan is \(a = a + e^{-a}\).

Deze vergelijking heeft geen oplossing, dus \((x_n)\) is niet begrensd.
\(\lim_{\nrightarrow \infty}\frac{x_{n+1}-x_{n}}{\ln(n+1)-\ln(n)}=\lim_{\nrightarrow \infty} \frac{e^{-x_{n}}}{\ln \frac{n+1}{n}}=\lim_{\nrightarrow \infty} \frac{ne^{-x_{n}}}{\ln \left(1+\frac{1}{n}\right)^n}=\lim_{\nrightarrow \infty} \frac{n}{e^{x_{n}}}\)
\(\lim_{\nrightarrow \infty} \frac{(n+1)-n}{e^{x_{n+1}}-e^{x_{n}}}=\lim_{\nrightarrow \infty} \frac{1}{e^{x_{n+1}}-e^{x_{n}}}=\lim_{\nrightarrow \infty} \frac{1}{e^{x_{n}}\left(e^{(x_{n+1}-x_{n})}-1\right)}=\lim_{\nrightarrow \infty} \frac{e^{-x_{n}}}{e^{e^{-x_{n}}}-1}= \lim_{x\rightarrow 0}\frac{x}{e^{x}-1} = 1\)

Re: limiet

\(\mbox{Zij }p_1, p_2, p_3, \cdots \)
de rij van priemgetallen.

Zo je weet geldt
\(\lim_{n \rightarrow \infty} \frac{p_n}{n \ln(n)} = 1\)
.
\(\mbox{Bereken }\lim_{n \rightarrow \infty}{(p_1 p_2 p_3 \cdots p_n)}^{\frac{1}{p_n}}\)

Re: limiet

kotje schreef:Bepaal:
\(\lim_{\nrightarrow\infty} ({\frac{1}{n+1}+\frac{1}{n+2}+ ... +\frac{1}{n+n})\)
\(\lim_{\nrightarrow\infty}\frac{1}{n+1}+\frac{1}{n+2}+ \cdots +\frac{1}{2n} = \lim_{\nrightarrow\infty}(\frac{1}{1}+\frac{1}{2}+ \cdots +\frac{1}{2n}) - (\frac{1}{1}+\frac{1}{2}+ \cdots +\frac{1}{n}) =\)

\(\lim_{\nrightarrow\infty}(\frac{1}{1}+\frac{1}{2} - \frac{1}{1}) + (\frac{1}{3}+\frac{1}{4} - \frac{1}{2}) + (\frac{1}{5}+\frac{1}{6} - \frac{1}{3}) + (\frac{1}{7}+\frac{1}{8} - \frac{1}{4}) + \cdots (\frac{1}{2n-1}+\frac{1}{2n} - \frac{1}{n}) =\)

\(\lim_{\nrightarrow\infty}\frac{1}{1} - \frac{1}{2} + \frac{1}{3}-\frac{1}{4}+ \frac{1}{5} - \frac{1}{6} + \frac{1}{7} - \frac{1}{8} + \cdots \frac{1}{2n-1} - \frac{1}{2n} = \ln(2) \)

Re: limiet

kotje schreef:Bepaal:
\(\lim_{n \rightarrow \infty} ({\frac{1}{n+1}+\frac{1}{n+2}+ ... +\frac{1}{n+n})\)
Toon aan dat als
\(\frac{p_n}{q_n} = \frac{1}{n+1}+\frac{1}{n+2}+ ... +\frac{1}{n+n}\)
\(p_n\)
deelbaar is door
\(3n+1\)
\(p_n\)
en
\(q_n\)
zijn hier uiteraard natuurlijke getallen.

Reageer